47
SARTHI – PUNE – UPSC – CET - 2019B Note: Answer key along with explanation for the answer is provided below, Objections if any for the answer of any question, can be sent to [email protected] from 17 th November, 2019 to 20 th November, 2019, under the subject line SARTHI – PUNE – UPSC – CET - 2019B- ANSWER KEY- QUERRY. No objection will be entertained if received without proper explanation/solution with the extract of relevant reference book or after 20 th November, 2019. GENERAL STUDIES PAPER – I - ANSWER KEY AND EXPLAINATION QUESTION NUMBER ANSWER QUESTION NUMBER ANSWER QUESTION NUMBER ANSWER 1 D 26 A 51 D 2 D 27 D 52 A 3 D 28 D 53 D 4 C 29 D 54 A 5 C 30 B 55 B 6 B 31 B 56 D 7 D 32 A 57 D 8 A 33 D 58 C 9 A 34 A 59 A 10 D 35 A 60 B 11 B 36 D 61 C 12 D 37 B 62 A 13 D 38 A 63 D 14 C 39 D 64 B 15 A 40 D 65 A 16 C 41 C 66 C 17 C 42 C 67 B 18 B 43 D 68 A 19 C 44 B 69 D 20 A 45 A 70 A 21 C 46 C 71 C 22 B 47 C 72 A 23 C 48 A 73 A 24 C 49 D 74 B 25 A 50 B 75 A

SARTHI PUNE UPSC CET - 2019B Objections if any for the ...sarthi-maharashtragov.in/sites/default/files/inline-files/SARTHI-PUNE-UPSC-CET-2019B...Syama Prasad Mukherjee (6 July 1901

  • Upload
    others

  • View
    306

  • Download
    31

Embed Size (px)

Citation preview

Page 1: SARTHI PUNE UPSC CET - 2019B Objections if any for the ...sarthi-maharashtragov.in/sites/default/files/inline-files/SARTHI-PUNE-UPSC-CET-2019B...Syama Prasad Mukherjee (6 July 1901

SARTHI – PUNE – UPSC – CET - 2019B

Note: Answer key along with explanation for the answer is provided below,

Objections if any for the answer of any question, can be sent to

[email protected] from 17th November, 2019 to 20th November, 2019,

under the subject line SARTHI – PUNE – UPSC – CET - 2019B- ANSWER KEY-

QUERRY.

No objection will be entertained if received without proper explanation/solution

with the extract of relevant reference book or after 20th November, 2019.

GENERAL STUDIES PAPER – I - ANSWER KEY AND EXPLAINATION

QUESTION NUMBER

ANSWER QUESTION NUMBER

ANSWER QUESTION NUMBER

ANSWER

1 D 26 A 51 D

2 D 27 D 52 A

3 D 28 D 53 D

4 C 29 D 54 A

5 C 30 B 55 B

6 B 31 B 56 D

7 D 32 A 57 D

8 A 33 D 58 C

9 A 34 A 59 A

10 D 35 A 60 B

11 B 36 D 61 C

12 D 37 B 62 A

13 D 38 A 63 D

14 C 39 D 64 B

15 A 40 D 65 A

16 C 41 C 66 C

17 C 42 C 67 B

18 B 43 D 68 A

19 C 44 B 69 D

20 A 45 A 70 A

21 C 46 C 71 C

22 B 47 C 72 A

23 C 48 A 73 A

24 C 49 D 74 B

25 A 50 B 75 A

Page 2: SARTHI PUNE UPSC CET - 2019B Objections if any for the ...sarthi-maharashtragov.in/sites/default/files/inline-files/SARTHI-PUNE-UPSC-CET-2019B...Syama Prasad Mukherjee (6 July 1901

Que. 1 d

C. Rajagopalachari, the veteran congress leader, prepared a formula for Congress league cooperation in

1944. It was a tacit acceptance of the League’s demand for Pakistan. Gandhi supported the formula. The

main points in the CR Plan were:

i. The League was to endorse the Indian demand for independence and to co-operate with the Congress

in formation of Provisional Interim Government for a transitional period.

ii. At the end of the War, a commission would be appointed to demarcate the districts having a Muslim

population in absolute majority and in those areas plebiscite to be conducted on all inhabitants

(including the non-Muslims) on basis of adult suffrage.

iii. All parties would be allowed to express their stance on the partition and their views before the

plebiscite.

iv. In the event of separation, a mutual agreement would be entered into for safeguarding essential

matters such as defense, communication and commerce and for other essential services.

v. The transfer of population, if any would be absolutely on a voluntary basis.

vi. The terms of the binding will be applicable only in case of full transfer of power by Britain to

Government of India.

Que. 2 d

Bose met Hitler and with his help, the ‘Freedom Army’ was formed which consisted of all the prisoners

of war of Indian origin captured by Germany and Italy. Dresden, Germany was made the office of the

freedom army. Bose came to be called ‘Netaji’ by the people of the Germany. He gave the famous

slogan, ‘Jai Hind’ from the free India, Germany. On October 21, 1943 Bose formed the provisional govt.

for free India at Singapore with S. C. Chatterjee , M.A. Aiyar, Lakshmi Swaminathan, etc. the famous

slogan, “Give me blood, I will give you freedom” was given in Malaya. On July 6, 1944 Bose addressed

Mahatma Gandhi as ‘Father of Nation’ from the Azad Hind Radio station (the first person to call Gandhi,

‘Father of Nation’).

Que. 3 d

The Republican Party of India is a political party in India. It has its roots in the Scheduled Castes

Federation led by B. R. Ambedkar. The 'Training School for Entrance to Politics' was established by

Ambedkar in 1956 which was to serve as an entry point to the Republican Party of India.

In June, 1951 Indian National Congress dissidents led by Jivatram Kripalani founded the KMPP. Its 2

leaders: Prafulla Chandra Ghosh and Tanguturi Prakasam, had been chief ministers of West Bengal and

Madras respectively. It contested the very first parliamentary elections in 1951 in India.

Syama Prasad Mukherjee (6 July 1901 – 23 June 1953) was an Indian politician, barrister and

academician, who served as the Minister for Industry and Supply in Prime Minister Jawaharlal Nehru's

cabinet. After falling out with Nehru, Mukherjee quit the Indian National Congress because of

differences of opinion with Nehru led government on the issue of Jammu & Kashmir. With the help of

Page 3: SARTHI PUNE UPSC CET - 2019B Objections if any for the ...sarthi-maharashtragov.in/sites/default/files/inline-files/SARTHI-PUNE-UPSC-CET-2019B...Syama Prasad Mukherjee (6 July 1901

the Rashtriya Swayamsevak Sangh, he founded the Bharatiya Jana Sangh, the predecessor to the

Bharatiya Janata Party, in 1951.

Que . 4 c

Fazl Ali Commission

Chairman : Fazl Ali

Members: Hridaynath Kunzru and K.M.Panikkar

After the formation of Andhra Pradesh on the basis of linguistic factor, all hell breaks loose. The other

regions also started demanding for creation of separate states on the basis of linguistic factor. The

intense pressure forced the Indian Government to form a new commission to visit the whole question of

whether the linguistic basis of separation of states can be considered or not. It led to the formation of

Fazl Ali Commission in December 1953.

The commission submitted its report on September 1955 and acknowledged 4 major factors to be taken

into account in any scheme of reorganization of states:

1. Linguistic and Cultural homogeneity.

2. Preservation and strengthening of the unity and security of the nation.

3. Financial, Economic and Administrative considerations.

4. Planning and promotion of the welfare of the people in each state as well as of the Nation as a whole.

It suggested the reorganization of 27 states of various categories into 16 states and 3 Union Territories.

The recommendations of the Fazl Ali Commission were accepted by the Indian Government with minor

modifications. As a result, the State Reorganization Act of 1956 was passed by the Parliament to give

effect to the recommendation of the commission. It led to the formation of 14 states and 6 Union

Territories on 1st November, 1956.

Que. 5 c

The Instrument of Accession is a legal document executed by Maharaja Hari Singh, ruler of the princely

state of Jammu and Kashmir, on 26 October 1947. By executing this document under the provisions of

the Indian Independence Act 1947, Maharaja Hari Singh agreed to accede to the Dominion of India.

John Mathai presented the first Railway Budget for independent India in November 1947.

Mahatma Gandhi was assassinated on 30 January 1948 in the compound of Birla House (now Gandhi

Smite), a large mansion in New Delhi.

The Reserve Bank of India was nationalized with effect from 1st January, 1949 on the basis of the

Reserve Bank of India (Transfer to Public Ownership) Act, 1948. All shares in the capital of the Bank were

deemed transferred to the Central Government on payment of a suitable compensation.

Que. 6 b

On December 31, 1929 at the Lahore (Pakistan, after independence)) Congress, Pandit Nehru hoisted

the Swaraj flag on the banks of the River Ravi and declared Purna Swaraj as the goal of freedom

movement with a fervent call to observe next January 26, 1930 as Independence Day.

Page 4: SARTHI PUNE UPSC CET - 2019B Objections if any for the ...sarthi-maharashtragov.in/sites/default/files/inline-files/SARTHI-PUNE-UPSC-CET-2019B...Syama Prasad Mukherjee (6 July 1901

The Lucknow session of Indian National Congress, presided over by a Moderate, Ambika Charan

Mujumdar finally readmitted the extremists led by Tilak to the congress fold. Another significant

development to take place at Lucknow was the coming together of the Muslim League and the Congress

and the presentation of common demands by them to the govt. this happened at a time when the

Muslim League now dominated by the younger militant nationalist, was coming closer to the Congress

objectives and turning increasingly anti-imperialist.

Que. 7 d

Rowlatt Act was based on the recommendations made to the Imperial Legislative Council by the Rowlatt

Commission, headed by the British Judge, Sir Sidney Rowlatt, to investigate the ‘seditious conspiracy’ of

the Indian people. ( The committee had recommended that activists should be deported or imprisoned

without trial for two years, and that even possession of seditious newspapers would be adequate

evidence of guilt).

1. The Act allowed political activists to be tried without juries or even imprisoned without trial.

2. It allowed arrest of Indians without warrant on the mere suspicion of ‘treason’. Such suspects could

be tried in secrecy without recourse to legal help.

3. A special cell consisting of three high court judges was to try such suspects and there was no court of

appeal above the panel. This panel could even accept evidence not acceptable under the Indian

Evidences Act.

4. The law of Habeas Corpus, the basis of civil liberty, was sought to be suspended.

Que. 8 a

Shreemati Nathibai Damodar Thackersey Women’s University (SNDT), Mumbai is the first Women’s

university in India as well as in South-East Asia. It was founded in 1916.

Que. 9 a

Zamindari Association, the reckoned to be the first political association of modern India. Formally

launched in Calcutta in March 1838, it was renamed the Landholders' Society shortly afterwards. Landed

magnates like Raja Radhakant dev, Dwarkanath Tagore, Prasanna Kumar Tagore, Rajkamal Sen and

Bhabani Charan Mitra were its leading members.

British Indian Association was founded on October 29, 1851 at Calcutta with Raja Radhakanta Dev and

Debendranath Tagore as its President and Secretary respectively. Other members of the Association

included Ramgopal Ghosh, Peary Chand Mitra and Krishnadas Pal. Its membership was kept exclusive to

Indians.

Indian National Association was the first declared Nationalist Organization founded in British India by

Surendranath Banerjee and Anand Mohan Bose in 1876 at Culcutta. It was originally established as

Bharat Sabha and held its first annual conference in Culcutta. It merged in INC in 1885.

The Poona Sarvajanik Sabha was founded in 1867 by Mahadev Govind Ranade and others in Bombay

with the object of serving as a bridge between the govt. and the people.

Page 5: SARTHI PUNE UPSC CET - 2019B Objections if any for the ...sarthi-maharashtragov.in/sites/default/files/inline-files/SARTHI-PUNE-UPSC-CET-2019B...Syama Prasad Mukherjee (6 July 1901

Que. 10 d

The statement - “The only hope of India is from the masses. The upper classes are physically and morally

dead.” - was made by Swami Vivekananda in the context of the need for mass action by the Indian

people.

Que. 11 b

Banskhera or Banskheda , meaning the land bearing Bamboo, is an ancient village in in Uttar Pradesh or

Bihar. Banskhera is known in the Indian history for the discovery of Banskhera copper plate of

Harshavardhana. The copper plate bears the signature the great king of kings, Harshavardhana. It was

issued in 22nd year of Harsha's reign i.e. 628-629 AD. This copper plate gives the ancestry of Harsha. It is

important that this plate bears signature of Harsha. This inscription was issued from place named

Vardhamanakoti.

The Madhuben plate inscription and the Sonpat inscription are also helpful to know the chronology of

Harsha.

Que. 12 d

According to the scriptures of all Buddhist schools, the first Buddhist Council was held soon after the

death of the Buddha, dated by the majority of recent scholars around 400 BCE, under the patronage of

the king Ajatashatru with the monk Mahakasyapa presiding, at Sattapanni caves Rajgriha (now Rajgir).

The Second Buddhist Council was held at Vaisali (or Vaishali), an ancient city in what is now the state of

Bihar in northern India, bordering Nepal under the patronage of King Kalasoka while it was presided by

Sabakami. This Council probably was held about a century after the first one, or about 383 BCE.

The Third Buddhist council was convened in about 250 BCE at Asokarama in Pataliputra, supposedly

under the patronage of Emperor Ashoka. This is however disputed, as mention of the council never

appears in the Edicts of Ashoka.

The fourth Buddhist council was held in 72 AD at Kundalvana, Kashmir. It was presided by Vasumitra,

while Asvaghosa was his deputy. The council was held under the patronage of Kushan King Kanishka of

Kushan Empire. Buddhism was divided into two sects namely Mahayan and Hinayan.

Que. 13 d

Storm Centres and Leaders of the Revolt.

Delhi – General Khan

Kanpur – Nana Saheb

Lucknow – Begum Hazrat Mahal

Bareilly – Khan Bahadur

Bihar – Kunwar Singh

Faizabad – Maulvi Ahmadullah

Jhansi – Rani Laxmibai

Page 6: SARTHI PUNE UPSC CET - 2019B Objections if any for the ...sarthi-maharashtragov.in/sites/default/files/inline-files/SARTHI-PUNE-UPSC-CET-2019B...Syama Prasad Mukherjee (6 July 1901

Que. 14 c

Konark Sun Temple is a 13th century Temple also known as the Black Pagoda built at Konark, in Odisha,

India. It was built by King Narasimha Deva of Eastern Ganga Dynasty around 1250.

Puri is one of the most popular pilgrimage destinations in Orissa and in India. The Jagannath Temple,

known as the White Pagoda, dates back to the 12th century. It is situated in a huge complex, and the 200

feet high temple dominates the Puri skyline. Puri is famous for its chariot festival or the rath yatra which

attracts thousands of pilgrims. Puri is a great center of Vaishnavism.

Que. 15 a

Buddhacharita ("Acts of the Buddha"; Buddhacaritam, Devanagari बुद्धचरितम्) is an epic poem in the

Sanskrit mahakavya style on the life of Gautama Buddha by Asvaghoṣa, composed in the early second

century CE.

Asvaghosa also wrote a Sanskrit Drama “Sariputra Prakaran” which deals about Sariputta or Sariputra

the disciple of Buddha.

He also wrote Saundarananda, a kavya poem with the theme of conversion of Nanda, Buddha's half-

brother, so that he might reach salvation. The first half of the work describes Nanda's life, and the

second half of the work describes Buddhist doctrines and ascetic practices.

Svapnavasavadattam is a Sanskrit play in six acts written by the ancient Indian poet Bhasa. It is probably

the best known of Bhasa's works.

Que. 16 c

Sattriya introduced by the great Vaishnava saint and reformer of Assam, Mahapurusha Sankaradeva.

Sattriya Nritya is accompanied by musical compositions called borgeets.

The costumes are usually made of pat – a silk produced in Assam which is derived from the mulberry

plant.

There are 2 types of costumes: the male costume comprising the dhoti and chadar and the female

costume comprising the ghuri and chadar.

Que. 17 c

Right knowledge, right faith, and right conduct are the three most essentials for attaining liberation in

Jainism. In order to acquire these, one must observe the five great principles:

Non-violence - Ahimsa

Truth - Satya

Non-stealing - Achaurya or Asteya

Celibacy/Chastity - Brahmacharya

Non-attachment/Non-possession – Aparigraha

Page 7: SARTHI PUNE UPSC CET - 2019B Objections if any for the ...sarthi-maharashtragov.in/sites/default/files/inline-files/SARTHI-PUNE-UPSC-CET-2019B...Syama Prasad Mukherjee (6 July 1901

Que. 18 b

The Battle of Bedara (1759) was fought between the English and Dutch army in Bengal, India. In this

battle, the Dutch force was decisively defeated by the British forces and that helped British East India

Company to assert their supremacy in India.

The Battle of Wandiwash was a decisive battle in India during the Seven Years' War. The Count de Lally's

army, burdened by a lack of naval support and funds, attempted to regain the fort at Vandavasi, now in

Tamil Nadu. He was attacked by Sir Eyre Coote's forces and decisively defeated.

Fort Saint George, citadel built by the British East India Company in Madras (now Chennai), India, later

becoming the British capital in south India. The fort, named in honour of Britain’s patron saint, is well

preserved by the state of Tamil Nadu (formerly Madras).The East India Company’s original trading

station in south India was at Masulipatam, established in 1611. It was moved to Madras, where

permission to build a fort was obtained from the raja of Chandragiri in 1639, mainly because it was

nearer the weaving centres from which the company obtained goods for export to Persia and the East

Indies. It became the headquarters of the company in south India in 1641, and it was the first company

settlement in India to be fortified.

The Treaty of Aix-la-Chapelle that ended the war made provision for Madras to be returned to the

British in exchange for Louisbourg in Acadia which had been captured by British forces in 1745. The

French besieged Madras again in 1759, this time without success.

Que. 19 c

Chauth (from Sanskrit meaning one-fourth) was a regular tax or tribute imposed, from early 18th

century, by the Maratha Empire in India. It was an annual tax nominally levied at 25% on revenue or

produce, hence the name. It was levied on the lands which were under nominal Mughal rule.

The sardeshmukhi was an additional 10% levy on top of the chauth. It is a tribute paid to the king. It was

started by Raja Som Shah of Ramnagar State.

Que. 20 a

Black Hole of Calcutta, scene of an incident on June 20, 1756, in which a number of Europeans were

imprisoned in Calcutta (now Kolkata) and many died. The Europeans were the remaining defenders of

Calcutta following the capture of the city by the nawab (ruler) Siraj al-Dawlah, of Bengal, and the

surrender of the East India Company’s garrison under the self-proclaimed governor of Bengal, John Z.

Holwell. The incident became a cause celebre in the idealization of British imperialism in India and a

subject of controversy.

Que. 21 c

According to this, any princely state under the direct or indirect (as a vassal) control of the East India

Company where the ruler did not have a legal male heir would be annexed by the company.

This was not introduced by Lord Dalhousie even though it was he who documented it, and used it widely

to acquire territories for the British.

As per this, any adopted son of the Indian ruler could not be proclaimed as heir to the kingdom. The

adopted son would only inherit his foster father’s personal property and estates.

Page 8: SARTHI PUNE UPSC CET - 2019B Objections if any for the ...sarthi-maharashtragov.in/sites/default/files/inline-files/SARTHI-PUNE-UPSC-CET-2019B...Syama Prasad Mukherjee (6 July 1901

The adopted son would also not be entitled to any pension that his father had been receiving or to any

of his father’s titles.

This challenged the Indian ruler’s long-held authority to appoint an heir of their choice.

Que. 22 b

On 6 October 1839, Debendranath Tagore established Tattvaranjini Sabha which was shortly thereafter

renamed the Tattwabodhini (Truth-seekers) Sabha. Initially confined to immediate members of the

Tagore family, in 2 years it mustered over 500 members.

The objective of Tattvabodhini Sabha was to encourage religious inquiries and disseminate the essence

of Upanishads. On 21 December 1841, Debendranath Tagore and his around 20 associates joined the

Brahmo Samaj and thus the mission of Raja Ram Mohan Roy got a fresh blood.

Que. 23 c

Vijayawada lies on the banks of Krishna River, covered by hills and canals. It is 18.5 km (11.5 mi) from

the state capital, Amravati.

Ahmadabad is located on the banks of the Sabarmati River, 30 km (19 mi) from the state capital

Gandhinagar, which is its twin city. Ahmedabad has emerged as an important economic and industrial

hub in India.

Srinagar is the summer capital of Jammu and Kashmir. It is situated in the Kashmir Valley and lies on the

banks of the Jhelum River, a tributary of the Indus. The city is famous for its gardens, lakes and

houseboats.

Varanasi, also called Benares, Banaras, or Kashi, city, southeastern Uttar Pradesh state, northern India. It

is located on the left bank of the Ganges (Ganga) River and is one of the seven sacred cities of Hinduism.

Que. 24 c

Coromandal Coast is the Southern part between Krishna and Kaveri Rivers.

Northern part of the Western Coast is called Konkan (Mumbai-Goa).

Chilka Lake is a brackish water lake along the Eastern Coastal plain.

Que. 25 a

Anshi National Park is located in a pristine part of the Western Ghats and is known for the richness of its

flora and fauna. It is located at a distance of 60 km from Karwar in Uttar Kannada district, Karnataka. The

park spans across an area of 340 sq. km and is believed to be India’s most endangered Wildlife Park.

Valmiki National Park, Tiger Reserve and Wildlife Sanctuary is located at the India-Nepal border in the

West Champaran district of Bihar, India on the bank of river Gandak. It is the only National park in Bihar.

The Orang National Park also known as Rajiv Gandhi Orang National Park which is located on the north

bank of the Brahmaputra River in the Darrang and Sonitpur districts of Assam, India, covers an area of

78.81 square kilometres.

Page 9: SARTHI PUNE UPSC CET - 2019B Objections if any for the ...sarthi-maharashtragov.in/sites/default/files/inline-files/SARTHI-PUNE-UPSC-CET-2019B...Syama Prasad Mukherjee (6 July 1901

Que. 26 a

The Tungabhadra project also known as Pampa Sagar is constructed across the Tungabhadra River, a

tributary of the Krishna River. The dam is in Hosapete, Ballari district of Karnataka.

Rihand project is the most important multi-purpose project in Uttar Pradesh. It is located in the borders

of Uttar Pradesh and Madhya Pradesh. The Rihand River flows across a narrow gorge in the Vindhyan

ranges in the Mirzapur district of Uttar Pradesh.

The Bheema irrigation project or Rajiv Bheema Project is a lift irrigation canal project located in

Mahbubnagar district in Telangana, India.

Tawa Reservoir is a reservoir on the Tawa River in central India. It is located in Itarsi of Hoshangabad

District of Madhya Pradesh state, above Betul district.

Que. 27 d

6 degree channel – separates Indira Point and Indonesia.

8 degree channel – separates Maldives and Minicoy.

9 degree channel – separates Lakshadweep and Minicoy islands.

10 degree channel – separates Andaman and Nicobar.

Que. 28 d

Methods like Solar winds, Degassing, Photosynthesis adds up in the modification and formation of

present atmosphere. All these methods helps in making the environment pollution free and healthy.

These methods also supports the survival of living beings.

One of the methods which is not related to the modification of the atmosphere is the method of

differentiation. There is no such method introduced yet.

Differentiation: The process by which earth forming material got separated into different layers is called

differentiation. Starting from the surface to the central parts, we have the layers like crust, mantle,

outer core and inner core. From the crust to the core, density of the material increases.

Que. 29 d

Mulching is a simple process of using garden leftovers such as straw, leaves, grass clippings and

shredded trees to cover any bare soil in the garden. Mulching returns organic materials and nutrients to

the soil and it has the added benefit of preventing weeds or unwanted grass growth. An ideal place for

mulch is around the base of trees and shrubs.

Advantages of mulching

The introduction of mulching into your garden will provide the following rewards:

• protection of the soil from harsh environmental conditions.

• prevention of weed growth.

• soil will retain more moisture.

Page 10: SARTHI PUNE UPSC CET - 2019B Objections if any for the ...sarthi-maharashtragov.in/sites/default/files/inline-files/SARTHI-PUNE-UPSC-CET-2019B...Syama Prasad Mukherjee (6 July 1901

• prevention of erosion.

• improved soil conditions and healthier plants.

• will encourage beneficial organisms such as earthworms to the soil.

• the mulch acts as a fertiliser. • less weeding and watering necessary

Some methods of soil conservation is listed below:

Intercropping: Different crops are grown in alternate rows and are sown at different times to protect the

soil from rain wash

Contour ploughing: Ploughing parallel to the contours of a hill slope to form a natural barrier for water

to flow down the slope. If ploughing in done at right angles to the hill slope, following the natural

contours of the hill, the ridges and furrows break the flow of water down the hill This prevents excessive

soil loss as gullies are less likely to develop and also reduce run-off so that plants receive more water.

Thus by growing crops in contour pattern, plants can absorb much of the rain water and erosion is

minimised. When viewed from above, the field looks like a contour map.

Shelter belt: In the coastal and dry regions, rows of trees are planted to check the wind movement to

protect the soil cover. Shelter belts are rows of trees, usually along fence lines.

Terrace farming: Terrace farming is the practice of cutting flat areas out of a hilly or mountainous

landscape in order to grow crops. Steps are cut out on the slopes of the hills making terraces. Crops are

then grown over it. Terrace farming reduces soil erosion.

Rock dam: Rocks are piled up across a channel to slow down the flow of water. This prevents gullies and

further soil loss

Mulching: The bare ground between plant is covered with a layer of organic matter like straw. It helps to

retain soil moisture.

Contour barrier: stones, grass, soil are used to build barrier along contours. Trenches are made in front

of the barriers to collect water. They intercept downslope flowing water and soil particles. These

barriers slow down the water movement and reduce its erosive force. They also filter out and trap many

of the suspended soil particles, keeping them from being washed out of the field.

Strip Cropping: Crops may be cultivated in alternate strips, parallel to one another. Some strips may be

allowed to lie fallow while in other different crops may be sown. Various crops are harvested at different

intervals. This ensures that at no time of the year the entire area is left bare or exposed. The tall growing

crops act as windbreaks and the strips which are often parallel to the contours help in increasing water

absorption by the soil by slowing down runoff.

Crop Rotation: Crop rotation is a practice in which a different crop is cultivated on a piece of land each

year. This helps to conserve soil fertility as different crops require different nutrients from the soil. Crop

rotation will provide enough time to restore lost nutrients.

Que. 30 b

Petrology: Petrology is a science that deals with the origin, history, occurrence, structure, chemical

composition, mineral composition, texture, alteration, relationship with other rocks, and classification of

rocks. It is the branch of geology that studies rocks and the conditions under which they form. Petrology

Page 11: SARTHI PUNE UPSC CET - 2019B Objections if any for the ...sarthi-maharashtragov.in/sites/default/files/inline-files/SARTHI-PUNE-UPSC-CET-2019B...Syama Prasad Mukherjee (6 July 1901

has three subdivisions: igneous, metamorphic, and sedimentary petrology. Igneous and metamorphic

petrology are commonly taught together because they both contain heavy use of chemistry, chemical

methods, and phase diagrams. Sedimentary petrology is, on the other hand, commonly taught together

with stratigraphy because it deals with the processes that form sedimentary rock.

Que.31 b

Census is the basis for reviewing the country's progress in the past decade, monitoring the on-going

schemes of the Government and most importantly, plan for the future. That is why the slogan of Census

2011 is "Our Census, Our Future".

The decennial Census of India has been conducted 15 times, as of 2011. While it has been undertaken

every 10 years, beginning in 1872 under british Viceroy Lord Mayo, the first complete census was taken

in 1881.[1] Post 1949, it has been conducted by the Registrar General and Census Commissioner of India

under the Ministry of Home Affairs, Government of India. All the censuses since 1951 were conducted

under the 1948 Census of India Act. The last census was held in 2011. The next will be held in 2021. Also

it was 1st conducted at the time of industrial revolutions too.

Census of India prior to Independence:

I. 1872 Census of india

II. 1881 Census of India

III. 1891 Census of India

IV. 1901 Census of India

V. 1911 Census of India

VI. 1921 Census of India

VII. 1931 Census of India

VIII. 1941 Census of India

Census of independent India

I. 1951 Census of India

II. 1961 Census of India

III. 1971 Census of India

IV. 1981 Census of India

V. 1991 Census of India

VI. 2001 Census of India

VII. 2011 Census of India

VIII. 2021 Census of India

Page 12: SARTHI PUNE UPSC CET - 2019B Objections if any for the ...sarthi-maharashtragov.in/sites/default/files/inline-files/SARTHI-PUNE-UPSC-CET-2019B...Syama Prasad Mukherjee (6 July 1901

Que.32 a

The Montreal Protocol on Substances that Deplete the Ozone Layer (a protocol to the Vienna

Convention for the Protection of the Ozone Layer) is an international treaty designed to protect the

ozone layer by phasing out the production of numerous substances that are responsible for ozone

depletion.

It was agreed on 26 August 1987, and entered into force on 16 September 1989, following a first

meeting in Helsinki, May 1989. Since then, it has undergone nine revisions, in 1990 (London), 1991

(Nairobi), 1992 (Copenhagen), 1993 (Bangkok), 1995 (Vienna), 1997 (Montreal), 1998 (Australia), 1999

(Beijing) and 2016 (Kigali).

As a result of the international agreement, the ozone hole in Antarctica is slowly recovering. Climate

projections indicate that the ozone layer will return to 1980 levels between 2050 and 2070. Due to its

widespread adoption and implementation it has been hailed as an example of exceptional international

co-operation, with Kofi Annan quoted as saying that "perhaps the single most successful international

agreement to date has been the Montreal Protocol". In comparison, effective burden sharing and

solution proposals mitigating regional conflicts of interest have been among the success factors for the

ozone depletion challenge, where global regulation based on the Kyoto Protocol has failed to do so. In

this case of the ozone depletion challenge, there was global regulation already being installed before a

scientific consensus was established. Also, overall public opinion was convinced of possible imminent

risks.

The two ozone treaties have been ratified by 197 parties (196 states and the European Union), making

them the first universally ratified treaties in United Nations history.

Kigali Agreement: Paris Agreement to mitigate climate change was followed by another major global

agreement – Kigali Agreement.

Kigali Agreement: Important Points

I. It is a legally binding agreement between the signatory parties with non-compliance measures.

II. It will come into effect from 1st January 2019.

III. It has shown a considerable flexibility in approach while setting phase-down targets for different

economies accommodating their developmental aspirations, different socio-economic compulsions, and

scientific & technological capabilities.

IV. It has divided the signatory parties into three groups-

1. The first group consists of rich and developed economies like USA, UK and EU countries who will

start to phase down HFCs by 2019 and reduce it to 15% of 2012 levels by 2036.

2. The second group consists of emerging economies like China, Brazil as well as some African

countries who will start phase down by 2024 and reduce it to 20% of 2021 levels by 2045.

3. The third group consists of developing economies and some of the hottest climatic countries like

India, Pakistan, Iran, Saudi Arabia who will start phasing down HFCs by 2028 and reduce it to 15% of

2024-2026 levels till 2047.

Page 13: SARTHI PUNE UPSC CET - 2019B Objections if any for the ...sarthi-maharashtragov.in/sites/default/files/inline-files/SARTHI-PUNE-UPSC-CET-2019B...Syama Prasad Mukherjee (6 July 1901

Que. 33 d

(i) National Waterway-1: Allahabad-Haldia stretch of the Ganga-Bhagirathi-Hooghly river

(1620 Km) declared as NW in 1986 in the states of Uttar Pradesh, Bihar, Jharkhand and

West Bengal.

ii) National Waterway-2: Sadiya-Dhubri stretch of the Brahmaputra river (891 Km) declared as

National Waterway in 1988 in the state of Assam.

iii) National Waterway-3: Kollam-Kottapuram stretch of West Coast Canal and Champakara and

Udyogmandal canals (205 Km) declared as National Waterway in 1993 in the state of Kerala

Page 14: SARTHI PUNE UPSC CET - 2019B Objections if any for the ...sarthi-maharashtragov.in/sites/default/files/inline-files/SARTHI-PUNE-UPSC-CET-2019B...Syama Prasad Mukherjee (6 July 1901

iv) National Waterway-4: Kakinada-Pudducherry stretch of canals and Kaluvelly tank, Bhadrachalam-

Rajahmundry stretch of river Godavari and Wazirabad-Vijayawada stretch of river Krishna (1078 Km)

declared as National Waterway in 2008 in the states of Andhra Pradesh, Tamil Nadu and Union Territory

of Puducherry.

v) National Waterway-5: Talcher-Dhamra stretch of Rivers, Geonkhali-Charbatia stretch of East Coast

Canal, Charbatia-Dhamra stretch of Matai river and Mahanadi Delta Rivers (588 Km) declared as

Page 15: SARTHI PUNE UPSC CET - 2019B Objections if any for the ...sarthi-maharashtragov.in/sites/default/files/inline-files/SARTHI-PUNE-UPSC-CET-2019B...Syama Prasad Mukherjee (6 July 1901

National Waterway in 2008 in the states of West Bengal and Orissa.

Que. 34 a

The land frontier of India is about 15200 KM. The neighbouring countries of India in decreasing order of

border length are : Bangladesh-China-Pakistan-Nepal-Myanmar-Bhutan-Afghanistan.

India is located in South Asia. It is largest country in South Asia in area and population. There are

seven neighbouring countries on the frontiers of India. 17 states of India have common land borders

with neighbouring countries.

Sr. No. Neighbouring country

Length of the common border ( in km)

No. of Indian states having common borders

Names of Indian states having common borders

Bangladesh 4096.7 5 West Bengal, Assam, Meghalaya, Tripura, Mizoram

China 3488 5 Jammu & Kashmir, Himachal Pradesh, Uttarakhand, Sikkim, Arunachal Pradesh

Pakistan 3323 4 Jammu & Kashmir, Punjab, Rajasthan, Gujarat

Nepal 1751 5 Uttarakhand, Uttar Pradesh, Bihar, West Bengal, Sikkim

Myanmar 1643 4 Arunachal Pradesh, Nagaland, Manipur, Mizoram

Bhutan 699 4 Sikkim, West Bengal, Assam, Arunachal Pradesh

Afghanistan 106 1 Jammu & Kashmir (POK)

Radcliffe Line-Border line between India and Pakistan, and India and Bangladesh. The border between

India and Pakistan was decided by Sir Cyril Radcliffe in 1947. Since Bangladesh was East Pakistan at that

time, the border between India and Bangladesh is still decided by the Radcliffe Line.

McMahon Line-Border line between India and China. This line was decided by Sir Henry McMahon in

1914

Page 16: SARTHI PUNE UPSC CET - 2019B Objections if any for the ...sarthi-maharashtragov.in/sites/default/files/inline-files/SARTHI-PUNE-UPSC-CET-2019B...Syama Prasad Mukherjee (6 July 1901

Durand Line-Border line between India and Afghanistan. This line was decided by Sir Henry Mortimer

Durand in 1896.

Since India and Pakistan both were called India at that time, the border line between Pakistan and

Afghanistan, at present is decided by the same line, i.e., Durand line. The border between India and

Afghanistan is very short. At present this line is between Pak Occupied Kashmir (POK) and Afghanistan.

In 1947, after independence Pakistan invaded the princely state of Jammu & Kashmir with the help of

local tribesmen and occupied some of its areas. Later on the princely state of Jammu & Kashmir

declared accession to India. India did not take back the occupied territory because the matter had been

referred to the UNO. Thus, this territory of Jammu & Kashmir which belongs properly to India, is still

under the control of Pakistan and is known as Pak Occupied Kashmir (POK).

Que. 35 a

Western Ghats are locally known by different names such as Sahyadri in Maharashtra, Nilgiri hills in

Karnataka and Tamil Nadu and Anaimalai hills and Cardamom hills in Kerala. Western Ghats are

comparatively higher in elevation and more continuous than the Eastern Ghats. Their average elevation

is about 1500m with the height increasing from North to South.

Annaimudi (2695m), the highest peak of Peninsular Plateau is located on the Anaimalai hills of the

Western Ghats.

Que. 36 d

E-type (Polar type) of climate as per Koeppen’s classification:

Higher areas of Jammu & Kashmir, Uttarakhand and Himachal Pradesh in which the temperature of the

warmest month varies from 0° to 10°C. Precipitation occurs in the form of snow.

Que. 37 b

Cabinet approves Umbrella scheme “Ocean Services, Technology, Observations, Resources, Modelling

and Science (O-SMART)” of Ministry of Earth Sciences.

Implementation of O-SMART will help in addressing issues relating to Sustainable Development Goal-14,

which aims to conserve use of oceans, marine resources for sustainable development.

This scheme (O-SMART) also provides necessary scientific and technological background required for

implementation of various aspects of Blue Economy.

Que. 38 a

The World Economic Outlook (WEO) is a report by the International Monetary Fund that analyzes key

parts of the IMF's surveillance of economic developments and policies in its member countries. It also

projects developments in the global financial markets and economic systems.

Que. 39 d

India Post Payments Bank (IPPB) is a 100% government owned payments bank operating under the

Department of Posts, Ministry of Communications, which aims to utilize all of India's 155,015 post

offices as access points and 3 lakh postal postmen and Grameen Dak Sewaks to provide house to house

banking services.

Page 17: SARTHI PUNE UPSC CET - 2019B Objections if any for the ...sarthi-maharashtragov.in/sites/default/files/inline-files/SARTHI-PUNE-UPSC-CET-2019B...Syama Prasad Mukherjee (6 July 1901

IPPB will offer demand deposits such as savings and current accounts up to a balance of Rs. 1 Lac,

digitally enabled payments and remittance services of all kinds between entities and individuals.

Provide access to third-party financial services such as insurance, mutual funds, pension, credit

products, forex, and more, in partnership with insurance companies, mutual fund houses, pension

providers, banks, international money transfer organizations, etc.

Issuance of ATM/Debit Cards. Payments banks, however, cannot issue credit cards.

Payments banks will be permitted to handle cross border remittance transactions in the nature of

personal payments or remittances on the current account.

Que. 40 d

The basic structure of Basel III remains unchanged with three mutually reinforcing pillars.

Pillar 1 : Minimum Regulatory Capital Requirements based on Risk Weighted Assets (RWAs) :

Maintaining capital calculated through credit, market and operational risk areas.

Pillar 2 : Supervisory Review Process : Regulating tools and frameworks for dealing with peripheral risks

that banks face.

Pillar 3 : Market Discipline : Increasing the disclosures that banks must provide to increase the

transparency of banks.

Que. 41 c

A regressive tax is a tax applied uniformly, taking a larger percentage of income from low-income

earners than from high-income earners. It is in opposition to a progressive tax, which takes a larger

percentage from high-income earners. A regressive tax affects people with low incomes more severely

than people with high incomes because it is applied uniformly to all situations, regardless of the

taxpayers. While it may be fair in some instances to tax everyone at the same rate, it is seen as unjust in

other cases.

Que. 42 c

On December 8, 1951, the Prime Minister Jawaharlal Nehru presented the first five-year plan to the

Parliament of India. This was based on the Harrod-Domar model.

Que. 43 d

The closed economy is self-sufficient, which means no imports come into the country and no exports

leave the country. The purpose of a closed economy is to provide domestic consumers with everything

they need from within the country's borders.

Que. 44 b

Sectors which come under the ' 100%' category are:

Agriculture & Animal Husbandry, Air-Transport Services (non-scheduled and other services under civil

aviation sector), Airports (Greenfield + Brownfield), Asset Reconstruction Companies, Auto-components,

Automobiles, Biotechnology (Greenfield), Broadcast Content Services (Up-linking & down-linking of TV

channels, Broadcasting Carriage Services, Capital Goods, Cash & Carry Wholesale Trading (including

Page 18: SARTHI PUNE UPSC CET - 2019B Objections if any for the ...sarthi-maharashtragov.in/sites/default/files/inline-files/SARTHI-PUNE-UPSC-CET-2019B...Syama Prasad Mukherjee (6 July 1901

sourcing from MSEs), Chemicals, Coal & Lignite, Construction Development, Construction of Hospitals,

Credit Information Companies, Duty Free Shops, E-commerce Activities, Electronic Systems, Food

Processing, Gems & Jewellery, Healthcare, Industrial Parks, IT & BPM, Leather, Manufacturing, Mining &

Exploration of metals & non-metal ores, Other Financial Services, Services under Civil Aviation Services

such as Maintenance & Repair Organizations, Petroleum & Natural gas, Pharmaceuticals, Plantation

sector, Ports & Shipping, Railway Infrastructure, Renewable Energy, Roads & Highways, Single Brand

Retail Trading, Textiles & Garments, Thermal Power, Tourism & Hospitality and White Label ATM

Operations.

Que. 45 a

The GDP deflator is a measure of the price level of all domestically produced final goods and services in

an economy. It is sometimes also referred to as the GDP Price Deflator or the Implicit Price Deflator. It

can be calculated as the ratio of nominal GDP to real GDP times 100 ([nominal GDP/real GDP]*100).

Que. 46 c

Deen Dayal Antyodaya Yojana (DAY) with an aim to uplift the urban poor folks by enhancing sustainable

livelihood opportunities through skill development. Keeping in view the objective of Make in India, Skill

Development is essential for socio economic betterment. Deen Dayal Antyodaya Yojana was launched

under the Ministry of Housing and Urban Poverty Alleviation (HUPA). Government of India has

provisioned Rs.500 crore for the scheme.

The scheme is integration of the National Urban Livelihoods Mission (NULM) and National Rural

Livelihoods Mission (NRLM).

National Urban Livelihoods Mission (NULM) is renamed as Deen Dayal Antyodaya Yojana-(DAY-NULM)

and in Hindi as - Rashtriya Shahri Aajeevika Mission. Under the scheme urban areas extends the

coverage to all the 4041 statutory cities and towns, there by covering almost the entire urban

population. Currently, all the urban poverty alleviating programmes covered only 790 towns and cities.

MAIN HIGHLIGHTS OF THE SCHEME:

Employment through Skill Training and Placement - An expenditure of Rs.15, 000 per person is allowed

on training of urban poor which is Rs.18, 000 in North-East and J&K. Moreover, training urban poor to

meet the enormous demand from urban citizens by imparting market-oriented skills through City

Livelihood Centers.

Social Mobilization and Institution Development - It will be done through formation of Self-Help

Groups (SHG) for training members and hand holding, an initial support of 10, 000 is given for each

group. Assistance of Rs.50, 000 is provided to Registered Area Level Federations.

Subsidy to urban poor - An interest subsidy of 5% - 7% for setting up individual micro-enterprises with a

loan of up to 2 lakh and for group enterprises with a loan limit of up to Rs.10 lakhs.

Shelters for urban homeless - Cost of construction of shelters for urban homeless is fully funded under

the Scheme.

Other means - Development of vendor markets and also the promotion of skills for the vendors through

setting up infrastructure and special projects for the rag picker and differently abled etc.

Page 19: SARTHI PUNE UPSC CET - 2019B Objections if any for the ...sarthi-maharashtragov.in/sites/default/files/inline-files/SARTHI-PUNE-UPSC-CET-2019B...Syama Prasad Mukherjee (6 July 1901

Que. 47 c

It appears that India’s finance ministry wants its youth and people well versed in financial and economic

terms before it launches its interim budget. It has started a string of tweets to educate the general

public on the budgetary process.

The ‘know your budget’ twitter series appears to be aimed at explaining the basic definitions of

budgetary terms.

Que. 48 a

Targeted at the idea of paperless governance, DigiLocker is a platform for issuance and verification of

documents & certificates in a digital way, thus eliminating the use of physical documents. Indian citizens

who sign up for a DigiLocker account get a dedicated cloud storage space that is linked to their Aadhaar

(UIDAI) number. Organizations that are registered with Digital Locker can push electronic copies of

documents and certificates (e.g. driving license, Voter ID, School certificates) directly into citizen’s

lockers. Citizens can also upload scanned copies of their legacy documents in their accounts. These

legacy documents can be electronically signed using the e-Sign facility.

Que. 49 d

Ductility - The property of metal by which it can be drawn into wires is called ductility.

Malleability – The property of metals by which they can be beaten into thin sheets is called malleability.

This is a characteristic property of metals.

The materials which generally possess these properties such as hardness, malleability, ductility,

sonorous and good conductors of heat and electricity are called metals. The examples of metals are iron,

copper, aluminium, calcium, magnesium, etc.

Que. 50 b

Mutualism – Both species benefit

Commensalism – One specie benefits and other is unaffected.

Que. 51 d

Large scale applications include its use as a precursor to forestry products, in colourless glass

production, and in electronics. Being the main compound of arsenic, the trioxide is the precursor to

elemental arsenic, arsenic alloys, and arsenide semiconductors.

A variety of applications exploit arsenic's toxicity, including the use of the oxide as a wood preservative.

Arsenic trioxide is a cancer medication that interferes with the growth and spread of cancer cells in the

body. Arsenic trioxide is used to treat a cancer of the blood and bone marrow called acute

promyelocytic leukaemia, or APL.

Que. 52 a

POSHAN Abhiyaan (National Nutrition Mission – NNM)

POSHAN Abhiyaan is a flagship programme of the ministry of Women and Child Development (MWCD),

Government of India, which ensures convergence with various programmes i.e.,Anganwadi Services,

Page 20: SARTHI PUNE UPSC CET - 2019B Objections if any for the ...sarthi-maharashtragov.in/sites/default/files/inline-files/SARTHI-PUNE-UPSC-CET-2019B...Syama Prasad Mukherjee (6 July 1901

Pradhan Mantri Matru Vandana Yojana ( PMMVY), Scheme for Adolescent Girls (SAG) of MWCD Janani

Suraksha Yojana (JSY), National Health Mission (NHM), Swachh-Bharat Mission, Public Distribution

System (PDS), Department Food & Public Distribution, Mahatma Gandhi National Rural Employment

Guarantee Scheme (MGNREGS) and Ministry of Drinking Water & Sanitation.

The goals of NNM are to achieve improvement in nutritional status of Children from 0-6 years,

Adolescent Girls, Pregnant Women and Lactating Mothers in a time bound manner during the next three

years beginning 2017-18.

It aims to reduce the level of stunting, under nutrition, anaemia and low birth weight babies.

Under POSHAN Abhiyaan, the government has fixed targets to reduce stunting, under nutrition,

anaemia and reduce low birth weight by 2%, 2%, 3%, and 2% per annum respectively.

Mission also strives to achieve reduction in stunting from 38.4% to 25% by 2022

Monitoring is done under POSHAN Abhiyaan through ICDS-CAS aimed to augment system strengthening

including improving the coverage and quality of nutrition services.

Que. 53 d

Poliovirus:

Poliomyelitis (polio) is highly infectious viral disease, which mainly affects young children below age of 5.

The virus is transmitted from person to person. After entering body, it multiplies in the intestine, from

where it can invade the nervous system and can cause paralysis. Its initial symptoms include fever,

fatigue, headache, vomiting, stiffness in the neck, and pain in the limbs. In some cases, it causes

permanent paralysis. There is no cure for polio; however it can only be prevented by immunization.

Que. 54 a

The central part of an atom is called nucleus. Particles present inside the nucleus are called nucleons

and they include mainly protons and neutrons. Due to the presence of protons nucleus has a positive

charge.

Que. 55 b

Cloning is the process of producing genetically identical individuals of an organism either naturally or

artificially. In nature, many organisms produce clones through asexual reproduction. Cloning in

biotechnology refers to the process of creating clones of organisms or copies of cells or DNA fragments

(molecular cloning)

Reproductive cloning generally uses "somatic cell nuclear transfer" (SCNT) to create animals that are

genetically identical. This process entails the transfer of a nucleus from a donor adult cell (somatic cell)

to an egg from which the nucleus has been removed, or to a cell from a blastocyst from which the

nucleus has been removed. If the egg begins to divide normally it is transferred into the uterus of the

surrogate mother. Such clones are not strictly identical since the somatic cells may contain mutations in

their nuclear DNA. Additionally, the mitochondria in the cytoplasm also contains DNA and during SCNT

this mitochondrial DNA is wholly from the cytoplasmic donor's egg, thus the mitochondrial genome is

not the same as that of the nucleus donor cell from which it was produced.

Page 21: SARTHI PUNE UPSC CET - 2019B Objections if any for the ...sarthi-maharashtragov.in/sites/default/files/inline-files/SARTHI-PUNE-UPSC-CET-2019B...Syama Prasad Mukherjee (6 July 1901

Dolly was the first mammal to have been successfully cloned from an adult somatic cell.

Que. 56 d

Mission Indradhanush is a health mission of the government of India. The ultimate goal of Mission

Indradhanush is to ensure full immunization with all available vaccines for children up to two years of

age and pregnant women. It aims to immunize all children under the age of 2 years, as well as all

pregnant women, against eight vaccine preventable diseases. The diseases being targeted are

diphtheria, whooping cough, tetanus, poliomyelitis, tuberculosis, measles, meningitis and Hepatitis B. In

addition to these, vaccines for Japanese encephalitis and Haemophilus influenzae type B are also being

provided in selected states. In 2016, four new additions have been made namely Rubella, Japanese

Encephalitis, Injectable Polio Vaccine Bivalent and Rotavirus. In 2017, Pneumonia was added to the

Mission by incorporating Pneumococcal conjugate vaccine under Universal Immunization Programme.

The special drive will focus on improving immunisation coverage in select districts and cities to ensure

full immunisation to more than 90% by December 2018.

Que. 57 d

Ozone molecules in the stratosphere are constantly being produced and destroyed by different types of

UV radiation from the sun. However, scientists have discovered that certain chemicals react with UV

radiation in the stratosphere, which causes them to break apart and release chlorine or bromine atoms.

These atoms, in turn destroy ozone molecules. A single chlorine atom can break apart more than

100,000 ozone molecules.

Ozone depleting substances include:

• chlorofluorocarbons (CFCs)

• hydrochlorofluorocarbons (HCFCs)

• hydrobromoflurocarbons (HBFCs)

• halons

• methyl bromide

• Methyl chloroform.

They have been used as:

• refrigerants in commercial, home and vehicle air conditioners and refrigerators

• foam blowing agents

• components in electrical equipment

• industrial solvents

• Fumigants.

As methyl bromide and halons are broken apart they release bromine atoms, which are 60 times more

destructive to ozone molecules than chlorine atoms.

Page 22: SARTHI PUNE UPSC CET - 2019B Objections if any for the ...sarthi-maharashtragov.in/sites/default/files/inline-files/SARTHI-PUNE-UPSC-CET-2019B...Syama Prasad Mukherjee (6 July 1901

Que. 58 c

ARTICLE 31A : SAVING OF LAWS PROVIDING FOR ACQUISITION OF ESTATES, ETC.

(1) Notwithstanding anything contained in article 13, no law providing for –

(a) The acquisition by the State of any estate or of any rights therein or the extinguishment or

modification of any such rights, or

(b) The taking over of the management of any property by the State for a limited period either in the

public interest or in order to secure the proper management of the property, or

(c) The amalgamation of two or more corporations either in the public interest or in order to secure the

proper management of any of the corporations, or

(d) The extinguishment or modification of any rights of managing agents, secretaries and treasurers,

managing directors, directors or managers of corporations, or of any voting rights of share-holders

thereof, or

(e) The extinguishment or modification of any rights accruing by virtue of any agreement, lease or

license for the purpose of searching for, or winning, any mineral or mineral oil, or the premature

termination or cancellation of and such agreement, lease or license, shall be deemed to be void on the

ground that it is inconsistent with, or takes away or abridges any of the rights conferred by article 14 or

article 19: Provided that where such law is a law made by the Legislature of a State, the provisions of

this article shall not apply thereto unless such law, having been reserved for the consideration of the

President, has received his assent:

Provided further that where any law makes any provision for the acquisition by the State of any estate

and where any land comprised therein is held by a person under his personal cultivation, it shall not be

lawful for the State to acquire any portion of such land as is within the ceiling limit applicable to him

under any law for the time being in force or any building or structure standing thereon or appurtenant

thereto, unless the law relating to the acquisition of such land, building or structure, provides for

payment of compensation at a rate which shall not be less than the market value thereof.

(2) In this article, –

(a) the expression “estate” shall, in relation to any local area, have the same meaning as that expression

or its local equivalent has in the existing law relating to land tenure in force in that area and shall also

include –

(i) Any jagir, inam or muafi or other similar grant and in the States of Tamil Nadu and Kerala, any

janmam right;

(ii) Any land held under ryotwari settlement;

(iii) Any land held or let for purposes of agriculture of for purposes ancillary thereto, including waste

land, forest land, land for pasture or sites of buildings and other structures occupied by cultivators of

land, agricultural laborers and village artisans;

(b) The expression “rights”, in relation to an estate, shall include any rights vesting in a proprietor, sub-

proprietor, under-proprietor, tenure-holder, raiyat, under-raiyat or other intermediary and any rights or

privileges in respect of land revenue.

Page 23: SARTHI PUNE UPSC CET - 2019B Objections if any for the ...sarthi-maharashtragov.in/sites/default/files/inline-files/SARTHI-PUNE-UPSC-CET-2019B...Syama Prasad Mukherjee (6 July 1901

ARTICLE 31B : VALIDATION OF CERTAIN ACTS AND REGULATIONS

Without prejudice to the generality of the provisions contained in article 31A, none of the Acts and

Regulations specified in the Ninth Schedule nor any of the provision thereof shall be deemed to be void,

or even to have become void, on the ground that such Act, Regulation or provision is inconsistent with,

or takes away or abridges any of the rights conferred by, any provisions of this part, and

notwithstanding any judgment, decree or order of any court or tribunal to the contrary, each of the said

Acts and Regulations shall, subject to the power of any competent Legislature to repeal or amend it,

continue in force.

ARTICLE 31C : SAVING OF LAWS GIVING EFFECT TO CERTAIN DIRECTIVE PRINCIPLES

Notwithstanding anything contained in article 13, no law giving effect to the policy of the State towards

securing all or any of the principles laid down in Part IV shall be deemed to be void on the ground that it

is inconsistent with, or takes away or abridges any of the rights conferred by article 14 or article 19; and

no law containing a declaration that it is for giving effect to such policy shall be called in question in any

court on the ground that it does not give effect to such policy:

Provided that where such law is made by the Legislature of a State, the provisions of this article shall not

apply thereto unless such law, having been reserved for the consideration of the President, has received

his assent.

Que. 59 a

Some of its recommendations were -

a) 3-tier system of Panchayati Raj should be replaced by the 2-tier system.

b) That would include zila parishad at the district level, and mandal panchayat consisting of a group of

villages having a total population of 15,000 to 20,000.

c) A district should be the first point for decentralization under popular supervision below the state

level.

d) Zila parishad should be the executive body.

e) It should be made responsible for planning at the district level.

f) Political parties should officially participate at all levels of Panchayat elections.

g) Panchayati Raj institutions should have compulsory powers of taxation so as to mobilize their own

financial resources.

h) Regular social audit by a district level agency and by a committee of legislators to check proper use of

funds allotted for the vulnerable social and economic groups.

i) The state government should not supersede the Panchayati Raj institutions.

j) In case of super session, elections should be held within six months from the date of super session.

k) Nyayan panchayats should be separate bodies from that of development Panchayats.

Page 24: SARTHI PUNE UPSC CET - 2019B Objections if any for the ...sarthi-maharashtragov.in/sites/default/files/inline-files/SARTHI-PUNE-UPSC-CET-2019B...Syama Prasad Mukherjee (6 July 1901

l) Nyaya panchayats should be presided over by a qualified judge.

m) Organizing and conducting the Panchayati Raj elections would be by the Chief Electoral Officer of a

state in consultation with the chief election commissioner.

n) Development functions should be transferred to the zila parishad.

o) All development staff should work under ZP’s control and supervision.

p) The voluntary agencies should play an important role in mobilizing the support of the people or

Panchayati Raj.

q) To look after the affairs of the Panchayati Raj institutions, a minister for Panchayati Raj should be

appointed in the state council of ministers.

r) Seats for SCs and STs should be reserved on the basis of their population.

s) Constitutional recognition to the Panchayati Raj institutions - This would give them the required status

and a guarantee of continuous functioning.

Que.60 b

Appointment and Term of office:

The President appoints the Attorney General (AG). The person who is appointed should be qualified to

be appointed a judge of the Supreme Court. That means, he should be a citizen of India and a judge of

some high court for five years or an advocate of some high court for ten years or should be an eminent

jurist, in the opinion of the president.

The constitution does not provide for fixed tenure to the AG. So, he holds office during the pleasure of

the president. He can be removed by the president at any time. There is no procedure or ground

mentioned in the constitution for his removal.

Following are the Rights of the AG:

(1) In the performance of his duties, he has right of audience in all courts in the territory of India.

(2) He has the right to speak or to take part in the proceedings of both the Houses of Parliament and

their joint sittings, but without a right to vote.

(3) He has the right to speak or to take part in the meeting of any committee of the Parliament of which

he is named as a member, but without a right to vote.

(4) He enjoys all the privileges and immunities that are available to a member of parliament.

Que. 61 c

The constitution of India establishes a federal system of government. It contains all the usual features of

federation, viz., two government, division of power, written constitution, supremacy of constitution,

division of powers, written constitution, supremacy of constitution, rigidity of constitution, independent

judiciary and bicameralism. However the Indian constitution also contains a large number of unitary or

non-federal features such as a strong centre, single constitution, single citizenship, flexibility of

constitution, integrated judiciary, appointment of state governor by the centre, all – india services,

emergency provisions, and so on.

Page 25: SARTHI PUNE UPSC CET - 2019B Objections if any for the ...sarthi-maharashtragov.in/sites/default/files/inline-files/SARTHI-PUNE-UPSC-CET-2019B...Syama Prasad Mukherjee (6 July 1901

Que. 62 a

The emergency provisions are contained in part XVIII of the constitution, from articles 352 to 360.

i. National Emergency on the ground of war or external aggression or armed rebellion is mentioned in

the article 352

ii. State emergency (President’s rule) on the ground of failure of constitutional machinery in the state is

mentioned in article 356 or failure to comply with the directions of the centre is mentioned in article

365 .

iii. Financial Emergency due to a threat to the financial stability or credit of India is mentioned in article

360. This type of emergency is never proclaimed in India.

During an Emergency, the Central government becomes all powerful and the states go into the total

control of the Centre. It converts the federal structure into a unitary one without a formal amendment

of the Constitution.

Difference between the national Emergency and president’s rule is given below:

S.N. National Emergency (352) President’s Rule (356)

1. It can be proclaimed only when the

security of India or a part of it is

threatened by war, external

aggression or armed rebellion.

It can be proclaimed when the government

of a state cannot be carried on in

accordance with the provisions of the

Constitution due to reasons which may not

have any connection with war, external

aggression or armed rebellion.

2. During its operation, the state

executive and legislature continue to

function and exercise the powers

assigned to them under

the Constitution. Its effect is that the

Centre gets concurrent powers of

administration and legislation in the

state.

During its operation, the state executive is

dismissed and the state legislature is either

suspended or dissolved. The president

administers the state through the governor

and the Parliament makes laws for the

state. In brief, the executive and

legislative powers of the state are assumed

by the Centre.

3. The Parliament can make laws on the

subjects enumerated in the State List

only by itself, i.e. it can’t delegate the

same to any other body or authority.

The Parliament can delegate the power to

make laws for the state to the President or

to any other authority specified by him. So

far, the practice has been for the president

to make laws for the state in consultation

with the members of Parliament from that

state.

4. There is no maximum period

prescribed for its operation. It can be

continued indefinitely with the

There is a maximum period prescribed for

its operation, that is, three years.

Thereafter, it must come to an end and the

Page 26: SARTHI PUNE UPSC CET - 2019B Objections if any for the ...sarthi-maharashtragov.in/sites/default/files/inline-files/SARTHI-PUNE-UPSC-CET-2019B...Syama Prasad Mukherjee (6 July 1901

approval of Parliament for every six

months.

normal constitutional machinery must be

restored in the state.

5. Under this, the relationship of the

Centre with all the states undergoes a

modification.

Under this, the relationship of only the

state under emergency with the Centre

undergoes a modification.

6. It affects fundamental rights (FR) of

the citizens.

It has no effect on Fundamental Rights

(FR) of the citizens.

7. Every resolution of Parliament

approving its proclamation or its

continuance must be passed by a

special majority.

Every resolution of Parliament approving

its proclamation or its continuance can be

passed only by a simple majority.

8. Lok Sabha can pass a resolution for

its revocation.

There is no such provision. It can be

revoked by the President only on his own

discretion.

Que. 63 d

President Ramnath Kovind has given the assent to the 124th constitutional Amendment Bill (which is

now Constitution 103rd amendment Act) providing 10 per cent reservation for economically weaker

sections.

Criterion for Reservation • People who have an annual income of less than Rs.8 lakhs, or

• People who own less than five acres of farm land, or

• People who have a house lesser than 1,000 sq feet in a town (or 100 sq yard in a notified municipal area).

Constitutional provisions of the Act:

The Act amends Articles 15 and 16 of the Constitution, by adding a clause which allows states to make

"special provision for the advancement of any Economically Weaker Section ( EWS) of citizens".

Article 15 prohibit the state from discriminating any citizen on ground of any religion, race, caste, sex,

place of birth or any of them. However, the government may make special provisions for advancement

of socially and educationally backwards or for SC/STs. The bill amend article 15 to additionally permit

the government to provide for the advancement of Economically Weaker Section ( EWS). It also said the

reservation would be "in addition to the existing reservations and subject to a maximum of 10 per cent

of the total seats in each category".

These "special provisions" would relate to "their admission to educational institutions, including private

educational institutions, whether aided or unaided by the state, other than the minority educational

institutions".

Article 16 prohibit discrimination in employment in any government office, However the government

can allow reservation for any “backward class of citizens”, if they are not adequately represented in the

services under the state. The bill amend article 16 to permit the government to reserve up to 10 % of all

posts for the Economically Weaker Sections (EWS) of citizens. The reservation of upto 10 % for

Page 27: SARTHI PUNE UPSC CET - 2019B Objections if any for the ...sarthi-maharashtragov.in/sites/default/files/inline-files/SARTHI-PUNE-UPSC-CET-2019B...Syama Prasad Mukherjee (6 July 1901

Economically Weaker Sections in educational institution and public employment will be in addition to

existing reservation.

Que. 64 b

Prime Minister – Part of both Planning commission and NITI Aayog

Chief Minister of all states: Part of NITI Aayog but not in planning commission

Lt. Governors of Union Territories: Part of NITI Aayog but not in planning commission

Chief Secretaries of all states: Not a part of NITI Aayog

The composition of NITI Aayog is as under:

a. Prime Minister of India as the Chairperson

Page 28: SARTHI PUNE UPSC CET - 2019B Objections if any for the ...sarthi-maharashtragov.in/sites/default/files/inline-files/SARTHI-PUNE-UPSC-CET-2019B...Syama Prasad Mukherjee (6 July 1901

b. Governing Council comprising the Chief Ministers of all the States and Lt.

Governors of Union Territories

c. Regional Councils will be formed to address specific issues and contingencies

impacting more than one state or a region. These will be formed for a specified

tenure. The Regional Councils will be convened by the Prime Minister and will

comprise of the Chief Ministers of States and Lt. Governors of Union Territories

in the region. These will be chaired by the Chairperson of the NITI Aayog or his

nominee.

d. Experts, specialists and practitioners with relevant domain knowledge as special

invitees nominated by the Prime Minister

e. The full-time organizational framework will comprise of, in addition to the Prime

Minister as the Chairperson:

i. Vice-Chairperson: To be appointed by the Prime Minister ii. Members: Full-time iii. Part-time members: Maximum of 2 from leading universities research

organizations and other relevant institutions in an ex-officio capacity. Part

time members will be on a rotational basis.

iv. Ex Officio members: Maximum of 4 members of the Union Council of

Ministers to be nominated by the Prime Minister. v. Chief Executive Officer : To be appointed by the Prime Minister for a

fixed tenure, in the rank of Secretary to the Government of India.

vi. Secretariat as deemed necessary.

The composition of the Planning Commission has been mentioned below:

(1) Prime Minister is the ex-officio Chairman of the commission. He presides over the meetings of the

commission.

(2) Deputy Chairman is the de-facto chairman of the commission. He is given the rank of a Cabinet

Minister. He is responsible to formulate and submit the draft Five Year Plan to central cabinet.

(3) The Finance Minister and the Planning Minister are the ex-officio members of the commission. In

addition, some other central ministers may be appointed as part-time members of the commission.

(4) The commission has four to seven full-time members who are experts in various fields such as

economics, industry, science and general administration. They enjoy the rank of a Minister of State.

(5) The commission has a member-secretary who is usually a senior IAS officer.

Planning Commission NITI Aayog

Enjoyed the powers to allocate funds To be an advisory body, or a think-

Page 29: SARTHI PUNE UPSC CET - 2019B Objections if any for the ...sarthi-maharashtragov.in/sites/default/files/inline-files/SARTHI-PUNE-UPSC-CET-2019B...Syama Prasad Mukherjee (6 July 1901

to ministries and state governments. tank. The powers to allocate funds might be vested in the finance ministry.

The last Commission had eight full-time members

The number of full-time members could be fewer than Planning Commission

States' role was limited to the National Development Council and annual interaction during Plan meetings.

State governments are expected to play a more significant role than they did in the Planning Commission.

Secretaries or member secretaries were appointment through the usual process

Secretaries to be known as the CEO and to be appointed by the prime minister.

Full Planning Commission had no provision for part-time members.

To have a number of part-time members, depending on the need from time to time.

The commission reported to National Development Council that had state chief ministers and lieutenant governors.

Governing Council has state chief ministers and lieutenant governors.

Had deputy chairperson, a member secretary and full-time members

New posts of CEO, of secretary rank, and Vice-Chairperson. Will also have five full-time members and two part-time members. Four cabinet ministers will serve as ex-officio members.

Policy was formed by the commission and states were then consulted about allocation of funds.

Consulting states while making policy and deciding on funds allocation. Final policy would be a result of that.

Had power to decide allocation of government funds for various programmes at national and state levels.

No power to allocate funds

Imposed policies on states and tied allocation of funds with projects it approved.

NITI is a think-tank and does not have the power to impose policies.

Que. 65 a

A Lokpal is an anti-corruption ombudsman who represents the public interest. The concept of an

ombudsman is borrowed from Sweden which was the world’s first country to introduce the

Ombudsmen concept.

Que. 66 c

Federalism is a system of government in which powers have been divided between the centre and its

constituent parts such as states or provinces. It is an institutional mechanism to accommodate two sets

of politics, one at the centre or national level and second at the regional or provincial level. Both the sets

of power are autonomous in their own spheres. A federal system is different from a unitary system in

that sovereignty is constitutionally split between two territorial levels so that each level can act

independently of each other in some areas.

Page 30: SARTHI PUNE UPSC CET - 2019B Objections if any for the ...sarthi-maharashtragov.in/sites/default/files/inline-files/SARTHI-PUNE-UPSC-CET-2019B...Syama Prasad Mukherjee (6 July 1901

There are two kinds of federations:

Holding Together Federation – in this type, powers are shared between various constituent parts to

accommodate the diversity in the whole entity. Here, powers are generally tilted towards the central

authority. Example: India, Spain, Belgium.

Coming Together Federation – in this type, independent states come together to form a larger unit.

Here, states enjoy more autonomy as compared to the holding together kind of federation. Example:

USA, Australia, Switzerland.

Features of the federal system

1. Governments at least two levels 2. Division of powers between various levels 3. Rigidity of constitution 4. Independence judiciary 5. Dual citizenship 6. Bicameralism

All federations might not have all the above features. Some of them may be incorporated depending

on what type of federation it is.

Federalism in India

India is a federal system but with more tilt towards a unitary system of government. It is sometimes

considered a quasi-federal system as it has features of both a federal and a unitary system. Article 1 of

the Indian Constitution states, ‘India, that is Bharat, shall be a union of states’. The word federation is

not mentioned in the constitution.

Elements of federalism were introduced into modern India by the Government of India Act of 1919

which separated powers between the centre and the provincial legislatures.

Federal features of the Indian Union

• Governments at two levels – centre and states

• Division of powers between the centre and states – there are three lists given in the Seventh Schedule of the Constitution which gives the subjects each level has jurisdiction in:

• Union List

• State List

• Concurrent List

• Supremacy of the constitution – the basic structure of the constitution is indestructible as laid out by the judiciary. The constitution is the supreme law in India.

• Independent judiciary – the constitution provides for an independent and integrated judiciary. The lower and district courts are at the bottom levels, the high courts are at the state levels and at the topmost position is the Supreme Court. All courts are subordinate to the Supreme Court.

Unitary features of the Indian Union

Page 31: SARTHI PUNE UPSC CET - 2019B Objections if any for the ...sarthi-maharashtragov.in/sites/default/files/inline-files/SARTHI-PUNE-UPSC-CET-2019B...Syama Prasad Mukherjee (6 July 1901

• The flexibility of the constitution – the constitution is a blend of flexibility and rigidity. Certain provisions of the constitution can be easily amended. In case the amendments seek to change aspects of federalism in India, the provision to bring about such amendments is not easy.

• More power vests with the Centre – the constitution guarantees more powers with the Union List. On the Concurrent List, the parliament can make laws that can override the laws made by a state legislature on some matters. The parliament can also make laws regarding certain subjects in the State List.

• Unequal representation of states in the Rajya Sabha – the representation of the states in the upper house is based on the states’ populations. For example, Uttar Pradesh has 31 seats and Goa, 1 in the Rajya Sabha. In an ideal federal system, all the states should have equal representation.

• The executive is a part of the legislature – in India, the executive in both the centre and the states is a part of the legislature. This goes against the principle of division of powers between the different organs of the government.

• Lok Sabha is more powerful than the Rajya Sabha – in our system, the lower house is more powerful than the upper house and unequal powers to two houses is against the principle of federalism.

• Emergency powers – the centre is provided with emergency powers. When an emergency is imposed, the centre has increased control over states. This undermines the autonomy of the states.

• Integrated judiciary – the judiciary in India is integrated. There is no separate judiciary at the centre and the state levels.

• Single citizenship – in India, only single citizenship is available to citizens. They cannot be citizens of the state as well. This helps in increasing the feeling of nationality as it forges unity amidst regional and cultural differences. It also augments fundamental rights such as the freedom of movement and residence in any part of the nation.

• Governor’s appointment – the governor of a state acts as the centre’s representative in the state. The state government does not appoint the governor, the centre does.

• New states formation – the parliament has the power to alter the territory of a state by increasing or reducing the area of the state. It can also change the name of a state.

• All India Services – through the All India Services such as the IAS, IPS, IRS, etc. the centre interferes in the executive powers of the states. These services also offer uniformity in administration throughout the nation.

• Integrated election machinery – the Election Commission of India is responsible for conducting free and fair elections at both the centre and the state levels in India. The members of the EC is appointed by the president.

• Veto over states bills – The governor of a state can reserve certain kinds of bills for the president’s consideration. The president enjoys absolute veto on these bills. He can even reject the bill at the second instance that is, when the bill is sent after reconsideration by the state legislature. This provision is a departure from the principles of federalism.

• Integrated audit machinery – the president of the country appoints the CAG who audits accounts of both the centre and the states.

• Power to remove key officials – the state government or state legislature does not have the authority to remove certain key government officials even at the state level like the election commissioner of a state, judges of the high courts, or the chairman of the state public service commissions.

Que. 67 b

Article 1 (3) says the territory of India comprises the following:

i. Territories of the states.

Page 32: SARTHI PUNE UPSC CET - 2019B Objections if any for the ...sarthi-maharashtragov.in/sites/default/files/inline-files/SARTHI-PUNE-UPSC-CET-2019B...Syama Prasad Mukherjee (6 July 1901

ii. Union territories and

iii. other territories that may be acquired by Government of India at any time.

The concept of union territories was not in the original version of the constitution, but was added by the

Constitution (Seventh Amendment) Act, 1956.

Que. 68 a

Liberty means the absence of restraints on the activities of individuals. and at the same time, provide

opportunities for the development of individual personalities.

The preamble secure to all citizen of India liberty of thought, expression, belief, faith and worship,

through their Fundamental Rights, it is enforceable in court of law, in case of violation.

Liberty is very essential for the successful functioning of the Indian democratic system. However, liberty

does not mean license to do what one likes, and has to enjoyed within the limitations mentioned in the

constitution. I.e, to say that the liberty conceived by the preamble or fundamental rights is not

absolute but qualified

The idea of Republic and the ideals of liberty, equality and fraternity as laid down in the

preamble to the constitution of India have been borrowed from French constitution.

Que. 69 d

Agriculture, including agricultural education and research, comes under State List. a, b and c are a part

of Concurrent List.

Que. 70 a

Union Minister Prakash Javadekar announced the approval for the new Gorewada International Zoo to

be inaugurated in Nagpur district of Maharashtra. bAccording to Forest Development Corporation of

Maharashtra Limited (FDCML), the Government of Maharashtra has already issued its Government

Resolution, for establishing International Standard Zoo and Rescue Centre at Gorewada village, Nagpur

District on 1914 hectares of Forest Land through FDCML.

Que. 71 c

PM- KISAN Scheme:

Pradhan Mantri Kisan Samman Nidhi (PM-KISAN) is a Central Sector scheme with 100% funding from

Government of India.

The Scheme is effective from 1.12.2018.

Under the Scheme an income support of Rs.6000/- per year is provided to all farmer families across the

country in three equal installments of Rs.2000/- each every four months.

Definition of family for the Scheme is husband, wife and minor children.

The entire responsibility of identification of beneficiary farmer families rests with the State / UT

Governments.

Page 33: SARTHI PUNE UPSC CET - 2019B Objections if any for the ...sarthi-maharashtragov.in/sites/default/files/inline-files/SARTHI-PUNE-UPSC-CET-2019B...Syama Prasad Mukherjee (6 July 1901

The fund is directly transferred to the bank accounts of the beneficiaries.

Farmers covered under the Exclusion Criteria of the Operational Guidelines are not eligible for the

benefit of the Scheme.

For enrollment, the farmer is required to approach the local patwari / revenue officer / Nodal Officer

(PM-Kisan) nominated by the State Government.

The Common Service Centres (CSCs) have also been authorized to do registration of the farmers for the

Scheme upon payment of fees.

Farmers can also do their self-registration through the Farmers Corner in the portal.

Farmers can also edit their names in PM-Kisan database as per their Aadhaar database / card through

the Farmers Corner in the portal.

Farmers can also know the status of their payment through the Farmers Corner in the portal.

Exclusion Categories:

The following categories of beneficiaries of higher economic status shall not be eligible for benefit under

the scheme.

(a) All Institutional Land holders.

(b) Farmer families in which one or more of its members belong to following categories

i) Former and present holders of constitutional posts

ii) Former and present Ministers/ State Ministers and former/present Members of LokSabha/

RajyaSabha/ State Legislative Assemblies/ State Legislative Councils,former and present Mayors of

Municipal Corporations, former and present Chairpersons of District Panchayats.

iii) All serving or retired officers and employees of Central/ State Government Ministries

/Offices/Departments and its field units Central or State PSEs and Attached offices /Autonomous

Institutions under Government as well as regular employees of the Local Bodies (Excluding Multi-Tasking

Staff /Class IV/Group D employees)

iv) All superannuated/retired pensioners whose monthly pension is Rs.10,000/-or more

(Excluding Multi-Tasking Staff / Class IV/Group D employees) of above category

v) All Persons who paid Income Tax in last assessment year

vi) Professionals like Doctors, Engineers, Lawyers, Chartered Accountants, and Architects registered with

Professional bodies and carrying out profession by undertaking practices.

Que.72 a

NITI Aayog has launched “Pitch to MOVE”.

Page 34: SARTHI PUNE UPSC CET - 2019B Objections if any for the ...sarthi-maharashtragov.in/sites/default/files/inline-files/SARTHI-PUNE-UPSC-CET-2019B...Syama Prasad Mukherjee (6 July 1901

“Pitch to MOVE” is organised by NITI Aayog in collaboration with Invest India and Society of Indian

Automobile Manufacturers (SIAM)

a mobility pitch competition that aims to provide budding entrepreneurs of India a unique opportunity

to pitch their business ideas to a distinguished jury.

"Pitch to MOVE aims to identify and incentivise the startups, which will help the Government realize its

vision of Shared, Connected, Intermodal and Environment Friendly Mobility for India. The objective is to

harness the latest disruption for generating employment and growth in our country.

The competition aims to identify and reward the start-ups offering innovative solutions for shared,

connected, and environment friendly mobility.

Que. 73 a

Joint Military Exercise MAITREE-2019 between India and Thailand will be conducted at Foreign Training

Node, Umroi (Meghalaya) from 16-29 Sep 2019.. Indian and Royal Thailand Army (RTA) comprising 50

soldiers each will participate in the exercise with an aim to share experience gained during various

counter terrorism operations in their respective countries. MAITREE is an annual training event which is

being conducted alternatively in Thailand and India since 2006.

Que. 74 b

Union Minister for Information and Broadcasting Prakash Javadekar flagged off a vehicle ‘Jaldoot’ to

spread the message of water conservation.Jaldoot, a travelling exhibition arranged by Pune-based

Regional Outreach Bureau of I&B ministry, will tour eight districts of Maharashtra in the next two

months. It will take the message of water conservation to the masses and highlight work done in this

connection by the Narendra Modi government in the first 100 days of its second term. Water was top

priority for the Centre and it had set up a separate ministry called Jalshakti for the purpose, adding that

it was committed to provide piped water to every household by 2024.

Que. 75 a

Bangladesh Prime Minister Sheikh Hasina conferred the Dr. Kalam Smriti International Excellence Award

2019 in Dhaka. The award citation lauded Prime Minister Hasina for her vision of a peaceful and

prosperous South Asia, free of tension, conflicts and terrorism. The award has been instituted in the

memory of former President Dr. A.P.J. Abdul Kalam. The award is given every year to honors statesmen

or leaders who have shown excellence in respective fields to achieve the best for their countries. The

Presidents of the Maldives, Ghana and Mauritius earlier received the prestigious award since its

introduction in 2015.

Page 35: SARTHI PUNE UPSC CET - 2019B Objections if any for the ...sarthi-maharashtragov.in/sites/default/files/inline-files/SARTHI-PUNE-UPSC-CET-2019B...Syama Prasad Mukherjee (6 July 1901

SARTHI – PUNE – UPSC – CET - 2019B

GENERAL STUDIES PAPER – II - ANSWER KEY AND EXPLAINATION

QUESTION NUMBER

ANSWER QUESTION NUMBER

ANSWER QUESTION NUMBER

ANSWER

1 D 21 D 41 B

2 A 22 A 42 A

3 B 23 A 43 C

4 A 24 D 44 C

5 A 25 A 45 B

6 C 26 D 46 A

7 B 27 C 47 A

8 A 28 D 48 B

9 D 29 B 49 B

10 A 30 B 50 B

11 D 31 C 51 D

12 D 32 B 52 A

13 D 33 D 53 B

14 C 34 B 54 A

15 B 35 C 55 A

16 D 36 A 56 B

17 A 37 C 57 A

18 D 38 D 58 A

19 D 39 D 59 A

20 A 40 D 60 B

Page 36: SARTHI PUNE UPSC CET - 2019B Objections if any for the ...sarthi-maharashtragov.in/sites/default/files/inline-files/SARTHI-PUNE-UPSC-CET-2019B...Syama Prasad Mukherjee (6 July 1901

QUE.1. (d)

Buying specialized products make us save our time as the example of American shows. Better quality of

products and more options are directly co-related. Hence, (d) is the correct option.

QUE.2. (a)

Free trade offers consumers best choices and it fosters competition. Hence, (a) is the correct option.

QUE.3. (b)

Example of Americans has been given to show that American strive for better quality and products and

more focused towards their standard of living. Hence, (b) is the correct option.

QUE.4. (a)

Food security promotes availability of food with adequate quantity, quality and nutrition. Thus, (1) is

correct. Food security is acceptable as per the given culture. Thus, (2) is incorrect. Hence, (a) is the

correct option.

QUE.5. (a)

Rights approach of food security puts pressure of public action. Thus, (a) is correct. Food court may not

be justiciable as per the passage. Thus, (b) is incorrect. Hence, (a) is the correct option.

QUE.6. (c)

Food security involves quality, quantity and nutrition of food products. Hence, (c) is correct option.

QUE.7. (b)

We cannot say that Right to food is more required for India or it is an offshoot or a byproduct of some

type of human rights. Similarly, we cannot co-relate that human rights are justiciable and right to food is

not. Hence, (b) is the correct option.

QUE.8. (a)

Second paragraph of passage tells poverty as a central reason for early marriage and paragraph three

mentions the consequences of early marriages, that how it thwarts the chances of education, and

adversely affect the process of growth & development of girl which can further lead to poverty, so it

forms a vicious circle. Hence (1) is right.

The last lines of Para 3 provides the fact that risk of death in pregnancy & delivery for girls under age 15

is five times higher than for women in their 20s. So (2) is incorrect.

Hence option (a) is correct.

Page 37: SARTHI PUNE UPSC CET - 2019B Objections if any for the ...sarthi-maharashtragov.in/sites/default/files/inline-files/SARTHI-PUNE-UPSC-CET-2019B...Syama Prasad Mukherjee (6 July 1901

QUE.9. (d)

Paragraph Three, clearly mentions that “poverty plays a central role in causing & perpetuating early

marriage”. But none of the option contains this point. So no one is correct answer in mentioned options.

Hence (d) is correct answer

QUE.10. (a)

As per the facts provided in passage, more than 60 million girls under the age of 18 are already married.

And if child marriage will continue at its present rate it is estimated that 100 million more girls will be

married in next decade. It means 160 million by 2030, so (a) is correct.

QUE.11. (d)

Second paragraph mentions that girls who are young married at age have an increased change of being

poor and remaining poor. Paragraph 3, mentions the high risk of death in case of girls below the age of

15 during pregnancy & delivery. And it also tells that how it put curb on development & growth of

women. Finally the concluding line of passage describes the inability of such society to progress where

child marriage is present. Hence (d) is correct option.

QUE.12 (d)

In second paragraph of passage meaning of “staying in programme” is given according to which

bureaucrat is supposed to think of his/her assigned task and how best to accomplish it. So as per

passage ‘staying in programme’ is related to accomplishment that is completion of task and not with

efficiency. So (d) is correct.

QUE.13. (d)

It is clearly mentioned in the passage that ‘bureaucratic mindset’ emerges from compartmentalised

environment so (1) is correct. Similarly in last paragraph it is written that compartmentalised

environment is structured in forms of rank & myriad. Which suggests about hierarchy model? Hence (3)

is also correct. But (2) is not correct as in third para only it is exclusively mentioned that bureaucracy

thinking must always be within box. Hence (d) is correct.

QUE.14. (c)

In third para of passage it is straightly written that ‘Max weber's’ model is “firmly ordered system of

superiority & subordination in which there is a supervision of lower officer by the higher officer”. This

line clearly meant that ‘Max weber’s’ theory of Bureaucracy promote hierarchy model. Hence (c) is

correct. The passage says that Bureaucrat is suppossed to think of his/her assigned taste and how best

to accomplish it. This means bureaucrats do not enjoy absolute authority. Also they have to complete

(accomplish) their task within prescribed format. So option (a) & (b) are incorrect.

Hence (c) is the correct option

Page 38: SARTHI PUNE UPSC CET - 2019B Objections if any for the ...sarthi-maharashtragov.in/sites/default/files/inline-files/SARTHI-PUNE-UPSC-CET-2019B...Syama Prasad Mukherjee (6 July 1901

QUE. 15-19

Order of speak:

I. Pratham → Prakash → Mayank

II. Prasunn → Vivek → Gautam

III. Sahil → Rajni → Prasunn

IV. Mayank → Sahil

I + II + III + IV:

Pratham → Prakash → Mayank → Sahil → Rajni → Prasunn → Vivek → Gautam

Time:

I. Prasunn < Prakash

II. Vivek < Mayank

III. Mayank < Rajni < Sahil

IV. Sahil < Prasunn

I + II + III + IV:

Vivek < Mayank < Rajni < Sahil < Prasunn < Prakash

QUE.15. (b) longest time duration for the speech was of Prakash.

QUE.16. (d) Gautam was spoke at the end.

QUE.17. (a) Pratham was spoken at the beginning.

QUE.18. (d) shortest duration of speech was of Vivek.

QUE.19. (d) Mayank was spoke first before Sahil.

QUE.20-24

Sunday D

Monday No Play

Tuesday B

D* Wednesday E

A* Thursday F

Friday C

Saturday A

QUE.20. (a) Monday

QUE.21. (d) Three

QUE.22. (a) Thursday

QUE.23. (a) A is the last play in the series.

QUE.24. (d) All required.

Page 39: SARTHI PUNE UPSC CET - 2019B Objections if any for the ...sarthi-maharashtragov.in/sites/default/files/inline-files/SARTHI-PUNE-UPSC-CET-2019B...Syama Prasad Mukherjee (6 July 1901

QUE.25. (a)

As written in the third paragraph, that Members of an organization may not internalized with the

organizational values. Thus, (1) is incorrect. In second paragraph it is written that values are what we

judge to be right. Thus, (2) is correct. (3) is correct as any organization cannot afford to have different

value's other than the individuals. Since, we are supposed to find out incorrect. Hence, (a) is the correct

option.

QUE.26. (d)

In second paragraph it is written that organizational vision must be consistent with organization core

values. Thus, (d) is correct. (a) and (c) are very specific leaders and decision makers are not alone

responsible for organizational values. (b) is only showing the problem not a solution. Hence, (d) is the

correct option.

QUE.27. (c)

Example of Army's organizational values shows that stated values are sometimes different than the

operating values and few people in the organization may not follow the organization values. Whenever

there is a discount between stated and operating value, it is difficult to determine 'acceptable' norm of

value. Hence, (c) is the correct option.

QUE.28. (d)

Passage is only showing the problem, not a single solution has been shared to avoid disconnect between

organizational values and individual's values. Hence, (d) is the correct option.

QUE.29. (b)

According to passage, social reform movements bring some changes which gained momentum through

Indian National Movement, when it became mass based. Hence, (b) is the correct option.

QUE.30. (b)

Impact of social change is not materialism but cross materialism. Thus, (1) is incorrect. (2) and (3)

directly given. Hence, (b) is the correct option.

QUE.31. (c)

In the last paragraph, it is written that particular social change will be evaluated in the light of the ideals

and change implies continuity. Thus, (c) is correct. (b) is not even the theme. (a) and (d) are very

ordinary as per the theme of the passage. Hence, (c) is the correct option.

QUE.32. (b)

Not every social change brings disturbance in the society. Thus, (3) is incorrect. (1) and (2) are directly

co-related with the passage theme. Hence, (b) is the correct option.

Page 40: SARTHI PUNE UPSC CET - 2019B Objections if any for the ...sarthi-maharashtragov.in/sites/default/files/inline-files/SARTHI-PUNE-UPSC-CET-2019B...Syama Prasad Mukherjee (6 July 1901

QUE.33. (d)

Passage is all about Mughal Era architecture and its special characteristics.

Thus, (d) is the correct option.

QUE.34. (b)

We cannot say through passages that whether domes were semi-circular or onion shaped, but mosaic

pattern were speciality of Mughal architecture. Thus, (2) and (3) are correct.

Hence, (b) is the correct option.

QUE.35. (c)

Jahangir’s quadrangle and figures of living beings were special features but not geometrical design.

Hence, (c) is the correct option.

QUE.36. (a)

Author is admiring Mughal Architecture, praising its artistic value.

Hence, (a) is the correct option.

QUE.37. (c)

We cannot say specifically that economic federalism is all about handling environment problem and that

too about by government as the authority. Thus, (1) ruled out. Passage is not discussing about any

consensus between authorities it only talks about distribution of powers between authority. Thus, (3)

ruled out. If local and sovereign powers are equally focussed so as to improve social welfare will act as

economic federalism. Similarly if environmental problem are handled in this way it would be called

environmental federalism.

Hence, (c) is the correct option.

QUE.38. (d)

Wealth of Nations endorses about the local and quality of local services and how it can maintained

properly. Similarly issues like defense should be taken care by sovereign bodies. Thus, wealth of nations

is all about distribution of power but not economic power. Hence, (d) is the correct answer.

QUE.39. (d)

Decentralized approach can be compromised if it is likely to improve social welfare or remove

inefficiency of decentralization. Over jurisdiction is the problem of decentralization not jurisdiction per

se.

Hence, (d) is the correct option.

Page 41: SARTHI PUNE UPSC CET - 2019B Objections if any for the ...sarthi-maharashtragov.in/sites/default/files/inline-files/SARTHI-PUNE-UPSC-CET-2019B...Syama Prasad Mukherjee (6 July 1901

QUE.40. (d)

M = 2W

M – 20 = W + 5

⇒ 2W – 20 = W + 5

W = 25

M = 50

Total = 75 Hence, (d) is the correct option.

QUE.41. (b)

Required ratio = 6 : 4 = 3 : 2

Hence, the correct option is (b).

QUE.42. (a)

If two kids, there are four possibilities: {(B, B), (B, G), (G, B), (G, G)}

But given one kid is girl, therefore possibilities are:

S = {(B, G) (G, B) (G, G)}

n(S) = 3

E = (G, G) ⇒ n(E) = 1

P(E) = n(E)/n(S) = 1/3

Hence, the correct option is (a).

Page 42: SARTHI PUNE UPSC CET - 2019B Objections if any for the ...sarthi-maharashtragov.in/sites/default/files/inline-files/SARTHI-PUNE-UPSC-CET-2019B...Syama Prasad Mukherjee (6 July 1901

QUE.43. (c)

29 + 31 + 30 + 31 + 31 + 30 + 31 + 25

1 odd days + 3 odd days + 2 odd days + 3 odd days + 3 odd days +

2 odd days + 3 odd days + 4 odd days = 21/7 odd days

* Remainder is 0.

Now, Monday = 0 Tuesday = 1 Wednesday = 2 Thursday = 3 Friday = 4

Saturday = 5 Sunday = 6

Therefore, Monday is the correct answer.

QUE.44. (c)

Because selling price is equal then,

% change = x + y + x y /100

= 10 + (–10) + 10(-10) /100

= - 1% i.e., 1% loss

Hence, the correct option is (c).

QUE.45. (b)

Let the highest score = x

and lowest score = x – 100

According to question,

40 × 28 + x + (x – 100) = 42 × 30

1120 + 2x – 100 = 1260

2x = 1260 – 1020

2x = 240

x = 120 Hence, the correct option is (b)

Page 43: SARTHI PUNE UPSC CET - 2019B Objections if any for the ...sarthi-maharashtragov.in/sites/default/files/inline-files/SARTHI-PUNE-UPSC-CET-2019B...Syama Prasad Mukherjee (6 July 1901

QUE.46. (a)

Given: A = 2/9 (B + C)

⇒ A/(B+C) = 2/9

A : (B+C) = 2:9

∴ A = 770 × (2/11) = 140

Hence, the correct option is (a).

QUE.47. (a)

Number of students study languages = 410 – 25 = 385

⇒ n (F ∪ S) = 385

⇒ n (F) = 180

⇒ n (S) = 240

⇒ n (F ∩ S) = ?

n (F ∪ S) = n (F) + n (S) – n (F ∩ S)

⇒ 385 = 180 + 240 – n (F ∩ S)

⇒ n (F ∩ S) = 420 – 385

n (F ∩ S) = 35

Hence, the correct option is (a).

Que.48. (b)

(1/2)×5 = 3

⇒ (1/3) ×5 = 2

By multiplying with 2 both sides,

⇒ 1/3 × 5 × 2 = 2 × 2

⇒ 1/3 × 10 = 4

Hence, the correct option is (b).

Page 44: SARTHI PUNE UPSC CET - 2019B Objections if any for the ...sarthi-maharashtragov.in/sites/default/files/inline-files/SARTHI-PUNE-UPSC-CET-2019B...Syama Prasad Mukherjee (6 July 1901

QUE.49 (b)

We know that, Profit (P) = SP – CP ............(i)

3P = 2SP – CP (According to question) ..............(ii)

3(SP – CP) = 2SP – CP

3SP – 3CP = 2SP – CP

SP = 2CP

P = 2CP – CP

P = CP or

SP = 2CP

If SP is double the CP then profit % is 100%.

Hence, (b) is the correct option.

QUE.50 (b)

Total routes connecting A to B is 6 and from B to C is 5.

There are two ways to go from B to C.

(i) Directly = 5 ways

(ii) B to A and then from A to C = 6 × 2 = 12 ways.

Total = 17 ways

Hence, (b) is the correct option.

QUE.51-53

Correct sequence would be

QUE.51. (d) F is sitting at extreme left end.

QUE.52. (a) B is sitting to the right of D.

QUE.53. (b) E is sitting between F and C.

Page 45: SARTHI PUNE UPSC CET - 2019B Objections if any for the ...sarthi-maharashtragov.in/sites/default/files/inline-files/SARTHI-PUNE-UPSC-CET-2019B...Syama Prasad Mukherjee (6 July 1901

QUE.54. (a)

It can be easily observed that all tigers are horses, tigers are not monkeys, at least one lion is a tiger and

monkeys are not tigers. Hence, (a) is the correct option.

QUE.55. (a)

OR

From above Venn diagrams, it is evident that some shoes are gloves (may/may not be). No pants are

shoes (may/may not be) and all gloves are shirts (may/may not be). But those shirts which are pants can

always be gloves. Hence, (a) is the correct option.

Page 46: SARTHI PUNE UPSC CET - 2019B Objections if any for the ...sarthi-maharashtragov.in/sites/default/files/inline-files/SARTHI-PUNE-UPSC-CET-2019B...Syama Prasad Mukherjee (6 July 1901

QUE.56. (b)

Number of students admitted in engineering = 400

⇒ (100 – 20) i.e., 80% of engineering students = 400

∴ Total number of engineering students = (400/80)× 100 = 500

Hence, the correct option is (b).

QUE.57. (a)

Value of 88% = 430

∴ Value of 1% = 430/88

∴ Value of 12% = (430/88) × 12 = 58.63 ≈ 59

Hence, the correct option is (a).

QUE.58. (a)

In Engineering:

= (400/80) × 100 = 500

In Commerce:

= (380/78) × 100 = 487.17 ≈ 487

In Science:

= (380/84) × 100 = 452.38 ≈ 452

In Arts:

= (400/82) × 100 = 487.8 ≈ 488

In Home Science:

= (430/88) × 100 = 488.6 ≈ 489

Hence, the correct option is (a).

Page 47: SARTHI PUNE UPSC CET - 2019B Objections if any for the ...sarthi-maharashtragov.in/sites/default/files/inline-files/SARTHI-PUNE-UPSC-CET-2019B...Syama Prasad Mukherjee (6 July 1901

Que.59. (a)

Let the population of the village in 2019 = x

According to question,

50000 /60000 = 60000/ x

5/ 6 = 60000/ x

x = 60000 × 6/5 = 72000

Hence, the correct option is (a).

QUE.60. (b)

Cost of motorcycle = Rs.54000

Paid amount = Rs.24000

Unpaid amount = 54000 – 24000 = Rs.30000

10% of 30000, 10% of 24000, . . . . . . ., 10% of 6000 = 3000, 2400, 1800, 1200, 600

Total interest = 3000 + 2400 + 1800 + 1200 + 600 = 9000

Total cost pay for motorcycle = 24000 + 5 × 6000 + 9000 = 24000 + 30000 + 9000 = Rs. 63000

Hence, the correct option is (b).